«

2009-03-23 Thread John White
I run lyx 1.6.2 on my slackware 12.1 (Vector Linux) system.  I compiled 
it from source.  I use the default CUA bindings which show that pushing 
the quote key (key with  on it will produce a .  However, it produces 
a «.  (if this does not come through, its two little ). If I switch to 
aqua bindings, the problem goes away.  However I see no way to edit the 
shortcuts to deal with this.


Any suggestions are appreciated. I have had this problem, if it be such, 
since, I believe, lyx 1.6.0.  I get around it by holding down the SHIFT 
key and then hitting the  key.  However, that is something of a bother.


John




Re: «

2009-03-23 Thread Vincent van Ravesteijn

John White schreef:
I run lyx 1.6.2 on my slackware 12.1 (Vector Linux) system.  I 
compiled it from source.  I use the default CUA bindings which show 
that pushing the quote key (key with  on it will produce a .  
However, it produces a «.  (if this does not come through, its two 
little ). If I switch to aqua bindings, the problem goes away.  
However I see no way to edit the shortcuts to deal with this.


Any suggestions are appreciated. I have had this problem, if it be 
such, since, I believe, lyx 1.6.0.  I get around it by holding down 
the SHIFT key and then hitting the  key.  However, that is something 
of a bother.


John



Do you know about the Document-Settings-Language-Quote Style option ?

Vincent


Re: «

2009-03-23 Thread John White
No, I did not.  That did the trick. 


Thank you very much!

John

Vincent van Ravesteijn wrote:

John White schreef:
I run lyx 1.6.2 on my slackware 12.1 (Vector Linux) system.  I 
compiled it from source.  I use the default CUA bindings which show 
that pushing the quote key (key with  on it will produce a .  
However, it produces a «.  (if this does not come through, its two 
little ). If I switch to aqua bindings, the problem goes away.  
However I see no way to edit the shortcuts to deal with this.


Any suggestions are appreciated. I have had this problem, if it be 
such, since, I believe, lyx 1.6.0.  I get around it by holding down 
the SHIFT key and then hitting the  key.  However, that is something 
of a bother.


John


Do you know about the Document-Settings-Language-Quote Style 
option ?


Vincent







Re: letter?

2009-03-23 Thread Jean-Marie Pacquet

Uwe Stöhr a écrit :

Jean-Marie Pacquet schrieb:

You should erase the starting comma in the options list of the Latex 
preamble (the one in front of footsepline=true). The new version of 
Koma-script scrlttr2 does not like it.


In LyX 1.6.x the template doesn't have this comma. What LyX version 
are you using Jean-Marie?


regards Uwe
I use various versions (1.4.2 to 1.6.1). The problem of commas in front 
of the options in the Koma-script letter2 template (as it is up to 1.5.4 
I believe) was brought to my attention when someone told me that the 
templates I uploaded to the wiki as french letters examples in July 2008 
were not  working (that was in october 2008).
So when Bruce wrote he had a problem with Koma options in a template, I 
immediatly thought of this. May be it's something different though...


regards
--
jean-marie


Re: letter?

2009-03-23 Thread Helge Hafting

bb wrote:

The Documentation User Guide from December 7, 2008 might be outdated?
The Description of writing a letter (p. 50 in the german version of the
document) does not describe the reality. I tried all types of
letter-documents, but none offers the described option of adress-right? 


For instance if I try letter(DIN-Brief, German) I get an environment for
an adressee, but I am unable to write the complete adress with name,
street and town? That is also true for the simple letter environment. 

You can't press enter because that will end the paragraph (and also the 
address paragraph type.)


But press ctrl+enter when you need seeveral lines in an address. That 
works. I have written many a letter in LyX. You should really use a 
template, as the letter document class is very fussy about getting most 
of the special paragraph types _and_ in the right order. with a 
template, you just fill out the fields and then it works well.


Helge Hafting


Re: Lilypond integration

2009-03-23 Thread Helge Hafting

Johannes Asal wrote:
I know that you can include Lilypond material by External Material. 
That's ok when you have few notation examples in your text and I 
appreciate that someone took the time to implement it. But I'm trying to 
write a book about jazz harmonics and therefore I need many musical 
examples that have to be changed often in the writing process. The main 
problems I have with the External Material approach are the following:


1. It interrupts the creative process, because you need to prepare the 
lilypond snippets in another editor and import them afterwards


I understand, as I have the same problem with figures. It is possible to 
ease the pain some, though. The external inset has a edit button, so 
you can click that in order to edit your music snippet. Newer versions 
of lyx doesn't have the button, instead you right-click the lilypond 
object and select edit externally from the menu. So you can get to 
it from LyX, but it will obviously still be an external application. 
You may have to define an editor for .ly files in the preferences.


2. You don't have a preview and therefore it is a bit difficult to see 
whether you have written something already or not


Tools-preferences-Graphics, then turn Instant Preview ON.
You should now get previews of all external insets, including lilypond 
music.  This is perhaps turned off by default, for performance reasons.


3. If you have to change one of the snippets, you have to find the 
corresponding file


Not really. right-click and select edit externally (Or, for older 
versions of lyx: click, then use the edit button in the dialog.)


I don't know if it's possible, but the ideal solution (for me) would be 
some kind of lilypond environment (somewhat like the math environment), 
where you can type in your lilypond code directly. After leaving this 
environment, the graphic is rendered and put into place in the LyX 
document. If you need to change it, you just click on the graphic and 
the code environment pops up again.


This is of course not a simple modification to do, and I guess it 
wouldn't be that fun either, but I'm sure it would be a valueable 
addition to LyX's feature list because as far as I know there is no 
solution today that combines text with music typesetting in a convenient 
and straightforward way. I would even volunteer for implementing parts 
of it if needed.


Support for lilypond-book would be interesting. And sure - if you 
volunteer to do some work then you will get it faster.


This process can be divided into several steps, you won't have to do it 
all at once.


1. Work on the support for lilypond-book.
   This will have LyX create latex output as usual,
   then, add code to run lilypond-book on the produced file
   Finally, let LyX run latex/pdflatex on the file produced, as usual.
   So this step is mostly about inserting an invocation
   of lilypond-book into the existing workflow.

   At this point, use a new document settings to decide whether
   lyx should do this lilypond-book step or not. Many lyx documents
   have no lilypond content, and many lyx users don't have lilypond.

   Use the latex inset for lilypond code in the beginning.
   To enter music, type stuff like
\begin{lilypond} { c d e } \end{lilypond}
   There will be no preview so far.

2. With lilypond-book working, start on the lilypond environment.
   first figure out if it ought to be a inset, a layout module, or
   whatever. I guess an inset will be the way.

   Then add the rule about how LyX produces latex code when processing
   a lilypond inset. This is easy - just output \begin{lilypond}
   followed by the inset contents, followed by \end{lilypond}

   At this point, you have working lilypond-book!

3. Some fine-tuning. Remove the document setting that specifies if this
   is a lilypond document. Instead, add logic so that LyX will run
   lilypond-book whenever the document contains a lilypond inset.
   You can perhaps make use of similiar logic that add latex packages
   on an as-needed basis.

4. The preview. Arrange so Lyx runs the inset content through
   lilypond, and display the resulting .png if this succeeds.
   I am not sure how to do this part, but look at how it is done
   for math and graphichs insets.


Helge Hafting


Re: Installing biblatex on OSX

2009-03-23 Thread jezZiFeR
Hello,

 I have a problem with installing bibtext on a second OSX-Computer. I put
 the
  biblatex.module-file directly into the …/library/Lyx-1.6-Folder. Maybe
 this
  is the problem, but I think this is what the wiki prompts me to do. I
  reconfigured Lyx various times without succes – it does not find the
 module.

 Did you put it in ~/Library/Application Support/LyX-1.6/layout folder?
 (That's where it belongs.)


Yes, that´s where I have put it…


  Running texhas I get error-messages for every file in the texmf-folder,
 for
  example:
 
  texhash: /usr/local/texlive/2008/texmf-var: directory not writable.
  Skipping...

 This indicates you need to run texhash as root: sudo texhash.


I did that now, but Lyx still does not find the biblatex-module.


 On my other computer I have a texmf-folder
  directly in the library, where the biblatex.module is saved, and
 everything
  works fine. But on the other computer this folder does not exist…

 It won't exist unless you create it.


Do I need that folder or is it okay, that biblatex-module is in
…/library/applicationsupport/lyx 1.6 ? Where do all the other files lie on
my computer, on which I don´t have that texmf-folfer directly in the
library?

Could I just copy that folder from one computer to the other?

Thank you, best
Jess


Re: Installing biblatex on OSX

2009-03-23 Thread BH
On Mon, Mar 23, 2009 at 10:40 AM, jezZiFeR jezzi...@googlemail.com wrote:
 Hello,

  I have a problem with installing bibtext on a second OSX-Computer. I put
 the
  biblatex.module-file directly into the …/library/Lyx-1.6-Folder. Maybe
 this
  is the problem, but I think this is what the wiki prompts me to do. I
  reconfigured Lyx various times without succes – it does not find the
 module.

 Did you put it in ~/Library/Application Support/LyX-1.6/layout folder?
 (That's where it belongs.)


 Yes, that´s where I have put it…

Then it should be recognized -- but only after you reconfigure LyX
(LyX  Reconfigure).

  Running texhas I get error-messages for every file in the texmf-folder,
 for
  example:
 
  texhash: /usr/local/texlive/2008/texmf-var: directory not writable.
  Skipping...

 This indicates you need to run texhash as root: sudo texhash.


 I did that now, but Lyx still does not find the biblatex-module.

texhash is relevant for updating your TeX installation when you add
new .sty or .cls packages. It is distinct from reconfiguring LyX,
which needs to be done when you add new .layout or .module files.

 On my other computer I have a texmf-folder
  directly in the library, where the biblatex.module is saved, and
 everything
  works fine. But on the other computer this folder does not exist…

 It won't exist unless you create it.


 Do I need that folder or is it okay, that biblatex-module is in
 …/library/applicationsupport/lyx 1.6 ? Where do all the other files lie on
 my computer, on which I don´t have that texmf-folfer directly in the
 library?

 Could I just copy that folder from one computer to the other?

.layout/.module files are OS agnostic: there is no special Linux or
Mac or Windows version of them. So you can copy them from one computer
to another so long as you put them in the right place. But your texmf
folder is *not* the right place for .layout/.module files; those
belong in your LyX User's directory (which on Mac is
~/Library/Application Support/LyX-1.6) in the layout subdir.

Please make sure when you're reporting your problems that you are
accurately describing what folders you are putting files in. There is
no .../library/applicationsupport/lyx 1.6 folder; it's
.../Library/Application Support/LyX-1.6 (note capitalization, spacing,
and hypthen). If you don't accurately describe it here, we're unsure
whether you're actually putting in in the right place or not, and so
unsure whether the problem is user error or a bug.

Bennett


Re: Lilypond integration

2009-03-23 Thread Piero Faustini

 Support for lilypond-book would be interesting. And sure - if you 
 volunteer to do some work then you will get it faster.

I would happily contribute, but I'm not a programmer. May I be of some help 
just giving some suggestion or testing?


 This process can be divided into several steps, you won't have to do it 
 all at once.

I think your 4-steps are a perfect roadmap.

I have a doubt, although it's regarding more the LaTeX-LilyPond integration: 
lily-book knows how to break music lines (or pages) in a given latex 
environment, but it can't guess if this environment will change after LaTeX 
compile. I mean: I'm writing a komascript book using some packages which can 
alter heavily the shape of the text, such as BibLaTeX, by adding more text. 
This is the reason why my LyX's LaTeX run up to 4 times when compiling, and 
everything works ok. Can lilypond-book give instructions at the beginning of 
this process? Can it be run more than once between Latex runs just to refine 
the process? If yes, what about LyX and all this mess?

Anyway, to all: keep me updated about any Lilypond-book integration attempt.
Piero



Re: Installing biblatex on OSX

2009-03-23 Thread jezZiFeR

Then it should be recognized -- but only after you reconfigure LyX

(LyX  Reconfigure).


I did that various times, but it still does not work.


Running texhas I get error-messages for every file in the texmf- 
folder,

for

example:


texhash: /usr/local/texlive/2008/texmf-var: directory not  
writable.

Skipping...


This indicates you need to run texhash as root: sudo texhash.



I did that now, but Lyx still does not find the biblatex-module.


texhash is relevant for updating your TeX installation when you add
new .sty or .cls packages. It is distinct from reconfiguring LyX,
which needs to be done when you add new .layout or .module files.


Thank you for that explanation!


On my other computer I have a texmf-folder

directly in the library, where the biblatex.module is saved, and

everything

works fine. But on the other computer this folder does not exist…


It won't exist unless you create it.



Do I need that folder or is it okay, that biblatex-module is in
…/library/applicationsupport/lyx 1.6 ? Where do all the other files  
lie on

my computer, on which I don´t have that texmf-folfer directly in the
library?

Could I just copy that folder from one computer to the other?


.layout/.module files are OS agnostic: there is no special Linux or
Mac or Windows version of them. So you can copy them from one computer
to another so long as you put them in the right place. But your texmf
folder is *not* the right place for .layout/.module files;


YES, that was it! I put it directly into the …/ly 1.6-folder! now it  
works!


On my other computer it really have this folder, which I do not have  
here. Is this folder used for LaTex? That´s the way it looks like:
inline: Bild 2.png




those
belong in your LyX User's directory (which on Mac is
~/Library/Application Support/LyX-1.6) in the layout subdir.

Please make sure when you're reporting your problems that you are
accurately describing what folders you are putting files in. There is
no .../library/applicationsupport/lyx 1.6 folder; it's
.../Library/Application Support/LyX-1.6 (note capitalization, spacing,
and hypthen). If you don't accurately describe it here, we're unsure
whether you're actually putting in in the right place or not, and so
unsure whether the problem is user error or a bug.


I see, thank you. Now I have put it in .../Library/Application Support/ 
LyX-1.6/layouts


Thank you a lot!
Jess*

Help for paper about LaTeX/LyX and the meaning of life

2009-03-23 Thread Piero Faustini
Hello,
 in a couple of months I have to speak at a conference about computers  
music/critical editions. I will introduce to a M$Word-enslaved audience the 
great advantages of WYSIWYM (by the way, I'm going to talk about music notation 
software LilyPond too, and the audience will be also Finale-enslaved: it's 
David vs. Goliat).

I'm a musicologist and I'm not a LaTeX/LyX pro so the thing would definitely be 
something much more like a divulgative/ads/propaganda thing than a specialized 
research. For this reason, in order to give the audience some content rather 
than advertising, I want to cite some statistics, relevant opinions, important 
projects/books/initiatives based on LyX and so on, but the LyX site lacks of 
all of this.

On the other hand, I would like to introduce the WYSIWYM “philosophy” but, as 
I'm not a semiologist, I don't know where to find some relevant thought (or 
some “effect quotation”!) on the whole Content-Form-Structure-and-the-meaning-
of-life stuff.

Any help would be apreciated and - if possible - referenced.

Thanks, Piero



Re: Help for paper about LaTeX/LyX and the meaning of life

2009-03-23 Thread Les Denham
On Monday 23 March 2009, Piero Faustini wrote:
 Any help would be apreciated and - if possible - referenced.

Piero,

Here are some on-line resources:
http://www.lyx.org/PressAboutLyX
http://www.linux.com/feature/56471
http://www.linuxjournal.com/article/9085


-- 
Les

~~
Please avoid sending me Word or PowerPoint attachments.
See http://www.gnu.org/philosophy/no-word-attachments.html


Re: letter?

2009-03-23 Thread Bruce Pourciau


On Mar 22, 2009, at 5:18 PM, Uwe Stöhr wrote:


Jean-Marie Pacquet schrieb:

You should erase the starting comma in the options list of the  
Latex preamble (the one in front of footsepline=true). The new  
version of Koma-script scrlttr2 does not like it.


In LyX 1.6.x the template doesn't have this comma. What LyX version  
are you using Jean-Marie?


regards Uwe


So in LyX 1.5.6 is there no way to view and use the Koma-script  
scrlttr2 template directly from within LyX?


Bruce

Dropping a document out of version control

2009-03-23 Thread Shahar Or
Dear list,

How is it possible to stop using version control on a document, please?

Many blessings.
-- 
שחר אור | 050-794 | http://www.shahar-or.co.il
   *** שיעורים פרטיים בלינוקס ותכנה חופשית ***


Selecting empty headings style still creates page numbers

2009-03-23 Thread Shahar Or
Dear list,

Perhaps I'm confused and doing it wrong.

I've selected empty for headings style but still there are page numbers
at the bottom of pages.

Many blessings.
-- 
שחר אור | 050-794 | http://www.shahar-or.co.il
   *** שיעורים פרטיים בלינוקס ותכנה חופשית ***


custom layout -- possible bug?

2009-03-23 Thread Ernesto Posse
Hello. I'm having a bit of an issue with a custom layout I'm trying to
create for ACM SIG proceedings (from the sig-alternate.cls available
at http://www.acm.org/sigs/publications/proceedings-templates)

First I'm creating a layoutfile named sig-alternate.layout and
placed it under my layouts directory. The file contains the following:

--- begin of sig-alternate.layout ---
#% Do not delete the line below; configure depends on this
#  \DeclareLaTeXClass[sig-alternate]{sig-alternate}

Format 11
Input stdclass.inc
--- end of sig-alternate.layout ---

And I create a sample file (test.lyx) with this layout. Then I
generate plain latex from it and this is what I obtain:

--- begin of test.tex ---
%% LyX 1.6.1 created this file.  For more info, see http://www.lyx.org/.
%% Do not edit unless you really know what you are doing.
\documentclass[english]{article}
\usepackage[T1]{fontenc}
\usepackage[latin9]{inputenc}

\usepackage{babel}

\begin{document}

\title{Blah blah}


\author{Me}

\maketitle

Something...
\end{document}
--- end of test.tex ---


The problem is that the first line has the wrong class (article)! Am I
doing something wrong? Why doesn't the generated latex begin with this
\documentclass{sig-alternate}?


Thanks.

PS: I'm running lyx 1.6.1 on ubuntu.

-- 
Ernesto Posse

Applied Formal Methods Group - Software Technology Lab
School of Computing
Queen's University - Kingston, Ontario, Canada


Re: letter?

2009-03-23 Thread Jean-Marie Pacquet

Bruce Pourciau a écrit :


On Mar 22, 2009, at 5:18 PM, Uwe Stöhr wrote:


Jean-Marie Pacquet schrieb:

You should erase the starting comma in the options list of the Latex 
preamble (the one in front of footsepline=true). The new version 
of Koma-script scrlttr2 does not like it.


In LyX 1.6.x the template doesn't have this comma. What LyX version 
are you using Jean-Marie?


regards Uwe


So in LyX 1.5.6 is there no way to view and use the Koma-script 
scrlttr2 template directly from within LyX?


Bruce


Just make sure that the Koma-script options list does not start with a 
comma. Hereunder is a part of the Latex preamble of Koma-script letter2 
template for an old Lyx version (1.4.2):

...
%% THE CLASS OPTIONS
%% Remove preceeding '%' to uncomment an item
\KOMAoptions{%
%,headsepline=true%separate the header with a line on page 1
%,footsepline=true% separate the footer with a line on page 1
%pagenumber=botcenter%   position of the page number (see docu)
%,parskip=false%  Use indent instead of skip (more options cf. docu)
,fromalign=center%alignment of the address
,fromrule=aftername%separate the address with a line?
,fromphone=true% print sender phone number
%,fromfax=true%  print sender fax number
,fromemail=true%   print sender e-mail address
,fromurl=true%   print sender URL
%,fromlogo=true% print a logo (position depends on fromalign)
%,addrfield=false%print an address field?
%,backaddress=false%  print the back address?
%,subject=afteropening,titled% alternative subject layout and position
%,locfield=narrow%  width of the (extra) location field
%,foldmarks=false%  print foldmarks?
%,numericaldate=true%  date layout
%,refline=wide% layout of the refline
}

You will notice that the first uncommented option 
(,fromalign=center%...) starts with a comma). If you try to use this 
template with a more recent Lyx version (in fact a more recent 
Koma-script version) you will get this error:


You have used \KOMAoptions to set `',
but KOMA-Script does not know any option named `'.
See the KOMA-Script manual for more informations about options and
their values.


The solution is to edit the Latex preamble (Menu Document-Parameters 
then Latex preamble) so that the first option reads fromalign=center% 
without the preceding comma.


I understand from Uwe that the new templates for Koma-script letter2 no 
longer have this preceding comma. May be you still have an old template 
with a new version of Koma-script.


Regards
--
jean-marie




Re: Installing biblatex on OSX

2009-03-23 Thread BH
On Mon, Mar 23, 2009 at 11:45 AM, jezZiFeR jezzi...@gmail.com wrote:
 texhash is relevant for updating your TeX installation when you add
 new .sty or .cls packages. It is distinct from reconfiguring LyX,
 which needs to be done when you add new .layout or .module files.

 Thank you for that explanation!


snip

 .layout/.module files are OS agnostic: there is no special Linux or
 Mac or Windows version of them. So you can copy them from one computer
 to another so long as you put them in the right place. But your texmf
 folder is *not* the right place for .layout/.module files;

 YES, that was it! I put it directly into the …/ly 1.6-folder! now it works!

 On my other computer it really have this folder, which I do not have here.
 Is this folder used for LaTex? That´s the way it looks like:

Yes -- everything under texmf is for TeX/BibTeX/LaTeX.

I suspect what confused you was the presence of the lyx folder in
~/Library/texmf/latex. That is not your lyx user's directory (which is
at ~/Library/Application Support/LyX-1.6) but is rather a directory
the LyX Installer creates for its own latex styles and classes. You
should not put anything in that directory, as it may get overwritten
by future versions of the installer.

Bennett


Re: Help for paper about LaTeX/LyX and the meaning of life

2009-03-23 Thread Steve Litt
On Monday 23 March 2009 12:07:00 pm Piero Faustini wrote:
 Hello,
  in a couple of months I have to speak at a conference about computers 
 music/critical editions. I will introduce to a M$Word-enslaved audience the
 great advantages of WYSIWYM (by the way, I'm going to talk about music
 notation software LilyPond too, and the audience will be also
 Finale-enslaved: it's David vs. Goliat).

 I'm a musicologist and I'm not a LaTeX/LyX pro so the thing would
 definitely be something much more like a divulgative/ads/propaganda thing
 than a specialized research. For this reason, in order to give the audience
 some content rather than advertising, I want to cite some statistics,
 relevant opinions, important projects/books/initiatives based on LyX and so
 on, but the LyX site lacks of all of this.

 On the other hand, I would like to introduce the WYSIWYM “philosophy”
 but, as I'm not a semiologist, I don't know where to find some relevant
 thought (or some “effect quotation”!) on the whole
 Content-Form-Structure-and-the-meaning- of-life stuff.

 Any help would be apreciated and - if possible - referenced.

 Thanks, Piero

Hi Piero,

If you haven't already used the word WYSIWYM in your title yet and 
advertised your talk as such, I'd personally use different terminology. I'd 
call it styles based authoring.

LyX is built from the bottom up to make it easy to use (not necessarily create 
or modify, but use) styles. Either character styles or paragraph styles 
(which we LyXers call environments).

Styles-based authoring is a must when writing a long document because it 
promotes consistency. I used the character style myEmph about 30 times last 
night, and every one of them looked identical, both in LyX and in the 
produced PDF. I didn't have to say to myself each time hey, in this book am 
I emphasizing by italicizing, bolding or both? I tell several stories in my 
book, and I like stories to look different from the rest of the text. I don't 
have to, with each new story, ask myself hey, did I italicize stories, 
indent them, shade them, put a box around them, or some combination? No, 
every time I tell a story, I just use my Story environment.

The end result is that my book has a consistency unmatched by people who don't 
use styles-based authoring. Be aware that you can do styles-based authoring 
with MS Word, WordPerfect, of if you're a masochist OpenOffice. But it's 
easier in LyX, and LyX also makes it harder to do one-off formatting of 
characters and paragraphs. We LyXers call such one-off formatting finger 
painting, and the results aren't very good. If one really needs to finger 
paint, that's probably a good indication that what's really needed is pixel 
editor or a vector graphics program.

Some people will tell you LyX is not WYSIWYG. All I know is it's WYSIWYG 
enough that I was able to proofread my book in LyX. A truly non-WYSIWYG would 
require regular recompilation to the finished form to proofread. Wordperfect 
4.x is a non-WYSIWYG example -- an even better one is HTML in a text editor.

Here are some of the reasons I personally use LyX:

* It's rock stable
* It does what you expect it to do
* It turns out VERY good looking text layout
* Its native format is simple to edit with an editor
* Its native format is simple to parse with a program
* Its native format is simple to create with a program
* It supports me with huge community of knowledgeable people
* When I finally write my math book, it will handle equations beautifully

Here are the disadvantages of LyX:

* Creation and modification of custom styles is much harder than MS Word or 
Wordperfect.

If you get ten or twenty more opinions, you'll have a great foundation for 
your presentation.

HTH

SteveT

Steve Litt
Recession Relief Package
http://www.recession-relief.US



Re: Selecting empty headings style still creates page numbers

2009-03-23 Thread M-L
On Mon, 23 Mar 2009 22:03:47 +0200
Shahar Or sha...@shahar-or.co.il wrote:

 Dear list,
 
 Perhaps I'm confused and doing it wrong.
 
 I've selected empty for headings style but still there are page
 numbers at the bottom of pages.
 
 Many blessings.

Try: Document/Settings/Page Layout/Page style = empty

HTH if not maybe a bit more information might make it clearer - eg.
operating system.

That will certainly help.
-- 
Registered Linux User:- 329524
***
The central benefit of Zen, in the context of ordinary ups and downs of
life, is not in preventing the minus and promoting the plus, but in
directing people to the fundamental reality that is not under the sway
of ups and downs. Muso Kokushi
(1275-1351)

***
Debian, just the best way to create magic
___


Re: Smart single quotes in LyX (1.6.1, Mac OS X 10.5.6)

2009-03-23 Thread Grant Jacobs


Thanks for your reply. This doesn't quite address what I was after: 
almost, but not quite.


In case it's of use to others I'm going to explain what I wanted 
again, then how I have a sort-of solution, or more accurately a 
solution that raises another problem in it's place.


I was asking for a means to set up LyX so that whenever I type a 
single quote ('), the single quote would be treated as a smart 
quote, i.e. LyX would interpret it as an open or close quote as 
appropriate.


(I was not trying to set some key sequence to generate a smart single 
quote, I already knew how to do that--that's actually what I wanted 
to *stop* having to do, a key sequence is already set for generating 
a smart single quote by default. I was trying to get a single quote 
to be treated as a smart quote without having to remember to type a 
special key sequence.)


Furthermore, what I really wanted was to still get a straight 
quote, using a key sequence. The overall effect wanted was that of 
having a key sequence that might by default generate a smart quote, 
generate a straight quote and have the quote character default to a 
smart quote. This can be achieved with the double quotes; I was 
trying to achieve the same with a single quote.



If you try set ' to be the mapped key in the fashion BH 
describes, LyX will complain that:


   Shortcut `'' is already bound to:
   self-insert
   You need to remove that binding before creating a new one.

These self-insert bindings seem to be set in a file held within the 
application bundle, so strictly speaking this solution should be 
out of bounds to an end user. Ideally what would nice was a means to 
have ' act as a smart quote through the user interface, and 
hopefully in a way I will not have keep manually updating the binding 
each time I update LyX. This doesn't seem to be possible at present.


My eventual sort-of solution was to edit the latinkeys.bind file (in 
the 'bind' directory of the Resources section of the application), so 
that


   \bind quoteright self-insert

now reads:

   \bind quoteright quote-insert single

This certainly makes all single quotes appear as smart quotes, 
except now I can't find a way to map cmd-' (or whatever) to generate 
a straight quote, as it insists on generating a quote-insert 
single!


While I suspect there will be a solution to this somehow, this does 
suggest to me that the system for applying character mappings has 
some weaknesses.



Grant


On Thu, Mar 19, 2009 at 9:53 PM, Grant Jacobs 
gjac...@bioinfotools.com wrote:


 Is there a means to make smart *single* quotes the default in the same way
 as is done for double quotes?


LyX  Preferences  Editing  Shortcuts

1. Enter quote in the Show key-bindings containing field
2. select quote-insert single
3. click the Modify button
4. click the Clear button on the dialog that pops up
5. type the key you want to bind smart single quotes to
6. click OK
7. click Save

Bennett



--
---
Grant Jacobs Ph.D. BioinfoTools
ph. +64 3 478 0095  (office, after 10am)   PO Box 6129,
or  +64 27 601 5917 (mobile)   Dunedin,
gjac...@bioinfotools.com   NEW ZEALAND.
   Bioinformatics tools: deriving knowledge from biological data  
Bioinformatics tools - software development - consulting - training

 18 years experience in bioinformatics ready to solve your problem
Check out the website for more details: http://www.bioinfotools.com


Re: Help for paper about LaTeX/LyX and the meaning of life

2009-03-23 Thread stefano franchi
2009/3/23 Steve Litt sl...@troubleshooters.com

 On Monday 23 March 2009 12:07:00 pm Piero Faustini wrote:
  Hello,
   in a couple of months I have to speak at a conference about computers 
  music/critical editions. I will introduce to a M$Word-enslaved audience
 the
  great advantages of WYSIWYM (by the way, I'm going to talk about music
  notation software LilyPond too, and the audience will be also
  Finale-enslaved: it's David vs. Goliat).
 
  I'm a musicologist and I'm not a LaTeX/LyX pro so the thing would
  definitely be something much more like a divulgative/ads/propaganda thing
  than a specialized research. For this reason, in order to give the
 audience
  some content rather than advertising, I want to cite some statistics,
  relevant opinions, important projects/books/initiatives based on LyX and
 so
  on, but the LyX site lacks of all of this.
 
  On the other hand, I would like to introduce the WYSIWYM â  philosophyâ
  but, as I'm not a semiologist, I don't know where to find some relevant
  thought (or some â  effect quotationâ  !) on the whole
  Content-Form-Structure-and-the-meaning- of-life stuff.
 
  Any help would be apreciated and - if possible - referenced.
 
  Thanks, Piero

 Hi Piero,

 If you haven't already used the word WYSIWYM in your title yet and
 advertised your talk as such, I'd personally use different terminology. I'd
 call it styles based authoring.

 LyX is built from the bottom up to make it easy to use (not necessarily
 create
 or modify, but use) styles. Either character styles or paragraph styles
 (which we LyXers call environments).

 Styles-based authoring is a must when writing a long document because it
 promotes consistency. I used the character style myEmph about 30 times last
 night, and every one of them looked identical, both in LyX and in the
 produced PDF. I didn't have to say to myself each time hey, in this book
 am
 I emphasizing by italicizing, bolding or both? I tell several stories in my
 book, and I like stories to look different from the rest of the text. I
 don't
 have to, with each new story, ask myself hey, did I italicize stories,
 indent them, shade them, put a box around them, or some combination? No,
 every time I tell a story, I just use my Story environment.

 The end result is that my book has a consistency unmatched by people who
 don't
 use styles-based authoring. Be aware that you can do styles-based authoring
 with MS Word, WordPerfect, of if you're a masochist OpenOffice. But it's
 easier in LyX, and LyX also makes it harder to do one-off formatting of
 characters and paragraphs. We LyXers call such one-off formatting finger
 painting, and the results aren't very good. If one really needs to finger
 paint, that's probably a good indication that what's really needed is pixel
 editor or a vector graphics program.

 Some people will tell you LyX is not WYSIWYG. All I know is it's WYSIWYG
 enough that I was able to proofread my book in LyX. A truly non-WYSIWYG
 would
 require regular recompilation to the finished form to proofread.
 Wordperfect
 4.x is a non-WYSIWYG example -- an even better one is HTML in a text
 editor.

 Here are some of the reasons I personally use LyX:

 * It's rock stable
 * It does what you expect it to do
 * It turns out VERY good looking text layout
 * Its native format is simple to edit with an editor
 * Its native format is simple to parse with a program
 * Its native format is simple to create with a program
 * It supports me with huge community of knowledgeable people
 * When I finally write my math book, it will handle equations beautifully

 Here are the disadvantages of LyX:

 * Creation and modification of custom styles is much harder than MS Word or
 Wordperfect.

 If you get ten or twenty more opinions, you'll have a great foundation for
 your presentation.

 HTH

 SteveT

 Steve Litt
 Recession Relief Package
 http://www.recession-relief.US


Hi Piero,

I'm following Steve's suggestion and posting another opinion.
I have been using LyX exclusively for the last 4 years, and I came to it
from a few years in Framemaker. I used Word before that (since version
1.0...), until it literally ate the first chapter of my dissertation. I have
next to zero coding abilities, and I am an academic in the humanities---a
skill set that may be pretty close to your audience's, I believe.

My reasons for using LyX:

* its file format is text-based and human-readable. That guarantees that my
files will be readable throughout my career---which I hope will last
another  few years. Closed-source formats like MS Word, Framemaker, etc.
expose you to the whims of their manufacturers. That's not so bad in the
private sector, when most of the documents produced are short-lived, but
it's crucial in the academia, when you may want to reuse documents, notes,
etcetera you have written 20 years ago or more. In my case: Adobe
discontinued Framemaker for Linux and for Mac  10.4 (it requires classic).
I now have to keep around (and 

child document not working as advertised?

2009-03-23 Thread Florin Oprina
Hi all.
I use Lyx 1.6.2. I created a child document, selected a default master
document and put all my custom commands in the preamble of the master file.
In the master file I included the child document.

Now, if I compile the child document the settings in the master file seem to
be ignored: the custom commands do not work and bibliographic references are
not resolved.

From the LyX wiki:``Render just a child document and LyX will make sure all
macros are defined correctly, even though their real definition is e.g. in
the master document.''

But I can't get LyX to work as advertised. What am I doing wrong?

Thanks.


«

2009-03-23 Thread John White
I run lyx 1.6.2 on my slackware 12.1 (Vector Linux) system.  I compiled 
it from source.  I use the default CUA bindings which show that pushing 
the quote key (key with  on it will produce a .  However, it produces 
a «.  (if this does not come through, its two little ). If I switch to 
aqua bindings, the problem goes away.  However I see no way to edit the 
shortcuts to deal with this.


Any suggestions are appreciated. I have had this problem, if it be such, 
since, I believe, lyx 1.6.0.  I get around it by holding down the SHIFT 
key and then hitting the  key.  However, that is something of a bother.


John




Re: «

2009-03-23 Thread Vincent van Ravesteijn

John White schreef:
I run lyx 1.6.2 on my slackware 12.1 (Vector Linux) system.  I 
compiled it from source.  I use the default CUA bindings which show 
that pushing the quote key (key with  on it will produce a .  
However, it produces a «.  (if this does not come through, its two 
little ). If I switch to aqua bindings, the problem goes away.  
However I see no way to edit the shortcuts to deal with this.


Any suggestions are appreciated. I have had this problem, if it be 
such, since, I believe, lyx 1.6.0.  I get around it by holding down 
the SHIFT key and then hitting the  key.  However, that is something 
of a bother.


John



Do you know about the Document-Settings-Language-Quote Style option ?

Vincent


Re: «

2009-03-23 Thread John White
No, I did not.  That did the trick. 


Thank you very much!

John

Vincent van Ravesteijn wrote:

John White schreef:
I run lyx 1.6.2 on my slackware 12.1 (Vector Linux) system.  I 
compiled it from source.  I use the default CUA bindings which show 
that pushing the quote key (key with  on it will produce a .  
However, it produces a «.  (if this does not come through, its two 
little ). If I switch to aqua bindings, the problem goes away.  
However I see no way to edit the shortcuts to deal with this.


Any suggestions are appreciated. I have had this problem, if it be 
such, since, I believe, lyx 1.6.0.  I get around it by holding down 
the SHIFT key and then hitting the  key.  However, that is something 
of a bother.


John


Do you know about the Document-Settings-Language-Quote Style 
option ?


Vincent







Re: letter?

2009-03-23 Thread Jean-Marie Pacquet

Uwe Stöhr a écrit :

Jean-Marie Pacquet schrieb:

You should erase the starting comma in the options list of the Latex 
preamble (the one in front of footsepline=true). The new version of 
Koma-script scrlttr2 does not like it.


In LyX 1.6.x the template doesn't have this comma. What LyX version 
are you using Jean-Marie?


regards Uwe
I use various versions (1.4.2 to 1.6.1). The problem of commas in front 
of the options in the Koma-script letter2 template (as it is up to 1.5.4 
I believe) was brought to my attention when someone told me that the 
templates I uploaded to the wiki as french letters examples in July 2008 
were not  working (that was in october 2008).
So when Bruce wrote he had a problem with Koma options in a template, I 
immediatly thought of this. May be it's something different though...


regards
--
jean-marie


Re: letter?

2009-03-23 Thread Helge Hafting

bb wrote:

The Documentation User Guide from December 7, 2008 might be outdated?
The Description of writing a letter (p. 50 in the german version of the
document) does not describe the reality. I tried all types of
letter-documents, but none offers the described option of adress-right? 


For instance if I try letter(DIN-Brief, German) I get an environment for
an adressee, but I am unable to write the complete adress with name,
street and town? That is also true for the simple letter environment. 

You can't press enter because that will end the paragraph (and also the 
address paragraph type.)


But press ctrl+enter when you need seeveral lines in an address. That 
works. I have written many a letter in LyX. You should really use a 
template, as the letter document class is very fussy about getting most 
of the special paragraph types _and_ in the right order. with a 
template, you just fill out the fields and then it works well.


Helge Hafting


Re: Lilypond integration

2009-03-23 Thread Helge Hafting

Johannes Asal wrote:
I know that you can include Lilypond material by External Material. 
That's ok when you have few notation examples in your text and I 
appreciate that someone took the time to implement it. But I'm trying to 
write a book about jazz harmonics and therefore I need many musical 
examples that have to be changed often in the writing process. The main 
problems I have with the External Material approach are the following:


1. It interrupts the creative process, because you need to prepare the 
lilypond snippets in another editor and import them afterwards


I understand, as I have the same problem with figures. It is possible to 
ease the pain some, though. The external inset has a edit button, so 
you can click that in order to edit your music snippet. Newer versions 
of lyx doesn't have the button, instead you right-click the lilypond 
object and select edit externally from the menu. So you can get to 
it from LyX, but it will obviously still be an external application. 
You may have to define an editor for .ly files in the preferences.


2. You don't have a preview and therefore it is a bit difficult to see 
whether you have written something already or not


Tools-preferences-Graphics, then turn Instant Preview ON.
You should now get previews of all external insets, including lilypond 
music.  This is perhaps turned off by default, for performance reasons.


3. If you have to change one of the snippets, you have to find the 
corresponding file


Not really. right-click and select edit externally (Or, for older 
versions of lyx: click, then use the edit button in the dialog.)


I don't know if it's possible, but the ideal solution (for me) would be 
some kind of lilypond environment (somewhat like the math environment), 
where you can type in your lilypond code directly. After leaving this 
environment, the graphic is rendered and put into place in the LyX 
document. If you need to change it, you just click on the graphic and 
the code environment pops up again.


This is of course not a simple modification to do, and I guess it 
wouldn't be that fun either, but I'm sure it would be a valueable 
addition to LyX's feature list because as far as I know there is no 
solution today that combines text with music typesetting in a convenient 
and straightforward way. I would even volunteer for implementing parts 
of it if needed.


Support for lilypond-book would be interesting. And sure - if you 
volunteer to do some work then you will get it faster.


This process can be divided into several steps, you won't have to do it 
all at once.


1. Work on the support for lilypond-book.
   This will have LyX create latex output as usual,
   then, add code to run lilypond-book on the produced file
   Finally, let LyX run latex/pdflatex on the file produced, as usual.
   So this step is mostly about inserting an invocation
   of lilypond-book into the existing workflow.

   At this point, use a new document settings to decide whether
   lyx should do this lilypond-book step or not. Many lyx documents
   have no lilypond content, and many lyx users don't have lilypond.

   Use the latex inset for lilypond code in the beginning.
   To enter music, type stuff like
\begin{lilypond} { c d e } \end{lilypond}
   There will be no preview so far.

2. With lilypond-book working, start on the lilypond environment.
   first figure out if it ought to be a inset, a layout module, or
   whatever. I guess an inset will be the way.

   Then add the rule about how LyX produces latex code when processing
   a lilypond inset. This is easy - just output \begin{lilypond}
   followed by the inset contents, followed by \end{lilypond}

   At this point, you have working lilypond-book!

3. Some fine-tuning. Remove the document setting that specifies if this
   is a lilypond document. Instead, add logic so that LyX will run
   lilypond-book whenever the document contains a lilypond inset.
   You can perhaps make use of similiar logic that add latex packages
   on an as-needed basis.

4. The preview. Arrange so Lyx runs the inset content through
   lilypond, and display the resulting .png if this succeeds.
   I am not sure how to do this part, but look at how it is done
   for math and graphichs insets.


Helge Hafting


Re: Installing biblatex on OSX

2009-03-23 Thread jezZiFeR
Hello,

 I have a problem with installing bibtext on a second OSX-Computer. I put
 the
  biblatex.module-file directly into the …/library/Lyx-1.6-Folder. Maybe
 this
  is the problem, but I think this is what the wiki prompts me to do. I
  reconfigured Lyx various times without succes – it does not find the
 module.

 Did you put it in ~/Library/Application Support/LyX-1.6/layout folder?
 (That's where it belongs.)


Yes, that´s where I have put it…


  Running texhas I get error-messages for every file in the texmf-folder,
 for
  example:
 
  texhash: /usr/local/texlive/2008/texmf-var: directory not writable.
  Skipping...

 This indicates you need to run texhash as root: sudo texhash.


I did that now, but Lyx still does not find the biblatex-module.


 On my other computer I have a texmf-folder
  directly in the library, where the biblatex.module is saved, and
 everything
  works fine. But on the other computer this folder does not exist…

 It won't exist unless you create it.


Do I need that folder or is it okay, that biblatex-module is in
…/library/applicationsupport/lyx 1.6 ? Where do all the other files lie on
my computer, on which I don´t have that texmf-folfer directly in the
library?

Could I just copy that folder from one computer to the other?

Thank you, best
Jess


Re: Installing biblatex on OSX

2009-03-23 Thread BH
On Mon, Mar 23, 2009 at 10:40 AM, jezZiFeR jezzi...@googlemail.com wrote:
 Hello,

  I have a problem with installing bibtext on a second OSX-Computer. I put
 the
  biblatex.module-file directly into the …/library/Lyx-1.6-Folder. Maybe
 this
  is the problem, but I think this is what the wiki prompts me to do. I
  reconfigured Lyx various times without succes – it does not find the
 module.

 Did you put it in ~/Library/Application Support/LyX-1.6/layout folder?
 (That's where it belongs.)


 Yes, that´s where I have put it…

Then it should be recognized -- but only after you reconfigure LyX
(LyX  Reconfigure).

  Running texhas I get error-messages for every file in the texmf-folder,
 for
  example:
 
  texhash: /usr/local/texlive/2008/texmf-var: directory not writable.
  Skipping...

 This indicates you need to run texhash as root: sudo texhash.


 I did that now, but Lyx still does not find the biblatex-module.

texhash is relevant for updating your TeX installation when you add
new .sty or .cls packages. It is distinct from reconfiguring LyX,
which needs to be done when you add new .layout or .module files.

 On my other computer I have a texmf-folder
  directly in the library, where the biblatex.module is saved, and
 everything
  works fine. But on the other computer this folder does not exist…

 It won't exist unless you create it.


 Do I need that folder or is it okay, that biblatex-module is in
 …/library/applicationsupport/lyx 1.6 ? Where do all the other files lie on
 my computer, on which I don´t have that texmf-folfer directly in the
 library?

 Could I just copy that folder from one computer to the other?

.layout/.module files are OS agnostic: there is no special Linux or
Mac or Windows version of them. So you can copy them from one computer
to another so long as you put them in the right place. But your texmf
folder is *not* the right place for .layout/.module files; those
belong in your LyX User's directory (which on Mac is
~/Library/Application Support/LyX-1.6) in the layout subdir.

Please make sure when you're reporting your problems that you are
accurately describing what folders you are putting files in. There is
no .../library/applicationsupport/lyx 1.6 folder; it's
.../Library/Application Support/LyX-1.6 (note capitalization, spacing,
and hypthen). If you don't accurately describe it here, we're unsure
whether you're actually putting in in the right place or not, and so
unsure whether the problem is user error or a bug.

Bennett


Re: Lilypond integration

2009-03-23 Thread Piero Faustini

 Support for lilypond-book would be interesting. And sure - if you 
 volunteer to do some work then you will get it faster.

I would happily contribute, but I'm not a programmer. May I be of some help 
just giving some suggestion or testing?


 This process can be divided into several steps, you won't have to do it 
 all at once.

I think your 4-steps are a perfect roadmap.

I have a doubt, although it's regarding more the LaTeX-LilyPond integration: 
lily-book knows how to break music lines (or pages) in a given latex 
environment, but it can't guess if this environment will change after LaTeX 
compile. I mean: I'm writing a komascript book using some packages which can 
alter heavily the shape of the text, such as BibLaTeX, by adding more text. 
This is the reason why my LyX's LaTeX run up to 4 times when compiling, and 
everything works ok. Can lilypond-book give instructions at the beginning of 
this process? Can it be run more than once between Latex runs just to refine 
the process? If yes, what about LyX and all this mess?

Anyway, to all: keep me updated about any Lilypond-book integration attempt.
Piero



Re: Installing biblatex on OSX

2009-03-23 Thread jezZiFeR

Then it should be recognized -- but only after you reconfigure LyX

(LyX  Reconfigure).


I did that various times, but it still does not work.


Running texhas I get error-messages for every file in the texmf- 
folder,

for

example:


texhash: /usr/local/texlive/2008/texmf-var: directory not  
writable.

Skipping...


This indicates you need to run texhash as root: sudo texhash.



I did that now, but Lyx still does not find the biblatex-module.


texhash is relevant for updating your TeX installation when you add
new .sty or .cls packages. It is distinct from reconfiguring LyX,
which needs to be done when you add new .layout or .module files.


Thank you for that explanation!


On my other computer I have a texmf-folder

directly in the library, where the biblatex.module is saved, and

everything

works fine. But on the other computer this folder does not exist…


It won't exist unless you create it.



Do I need that folder or is it okay, that biblatex-module is in
…/library/applicationsupport/lyx 1.6 ? Where do all the other files  
lie on

my computer, on which I don´t have that texmf-folfer directly in the
library?

Could I just copy that folder from one computer to the other?


.layout/.module files are OS agnostic: there is no special Linux or
Mac or Windows version of them. So you can copy them from one computer
to another so long as you put them in the right place. But your texmf
folder is *not* the right place for .layout/.module files;


YES, that was it! I put it directly into the …/ly 1.6-folder! now it  
works!


On my other computer it really have this folder, which I do not have  
here. Is this folder used for LaTex? That´s the way it looks like:
inline: Bild 2.png




those
belong in your LyX User's directory (which on Mac is
~/Library/Application Support/LyX-1.6) in the layout subdir.

Please make sure when you're reporting your problems that you are
accurately describing what folders you are putting files in. There is
no .../library/applicationsupport/lyx 1.6 folder; it's
.../Library/Application Support/LyX-1.6 (note capitalization, spacing,
and hypthen). If you don't accurately describe it here, we're unsure
whether you're actually putting in in the right place or not, and so
unsure whether the problem is user error or a bug.


I see, thank you. Now I have put it in .../Library/Application Support/ 
LyX-1.6/layouts


Thank you a lot!
Jess*

Help for paper about LaTeX/LyX and the meaning of life

2009-03-23 Thread Piero Faustini
Hello,
 in a couple of months I have to speak at a conference about computers  
music/critical editions. I will introduce to a M$Word-enslaved audience the 
great advantages of WYSIWYM (by the way, I'm going to talk about music notation 
software LilyPond too, and the audience will be also Finale-enslaved: it's 
David vs. Goliat).

I'm a musicologist and I'm not a LaTeX/LyX pro so the thing would definitely be 
something much more like a divulgative/ads/propaganda thing than a specialized 
research. For this reason, in order to give the audience some content rather 
than advertising, I want to cite some statistics, relevant opinions, important 
projects/books/initiatives based on LyX and so on, but the LyX site lacks of 
all of this.

On the other hand, I would like to introduce the WYSIWYM “philosophy” but, as 
I'm not a semiologist, I don't know where to find some relevant thought (or 
some “effect quotation”!) on the whole Content-Form-Structure-and-the-meaning-
of-life stuff.

Any help would be apreciated and - if possible - referenced.

Thanks, Piero



Re: Help for paper about LaTeX/LyX and the meaning of life

2009-03-23 Thread Les Denham
On Monday 23 March 2009, Piero Faustini wrote:
 Any help would be apreciated and - if possible - referenced.

Piero,

Here are some on-line resources:
http://www.lyx.org/PressAboutLyX
http://www.linux.com/feature/56471
http://www.linuxjournal.com/article/9085


-- 
Les

~~
Please avoid sending me Word or PowerPoint attachments.
See http://www.gnu.org/philosophy/no-word-attachments.html


Re: letter?

2009-03-23 Thread Bruce Pourciau


On Mar 22, 2009, at 5:18 PM, Uwe Stöhr wrote:


Jean-Marie Pacquet schrieb:

You should erase the starting comma in the options list of the  
Latex preamble (the one in front of footsepline=true). The new  
version of Koma-script scrlttr2 does not like it.


In LyX 1.6.x the template doesn't have this comma. What LyX version  
are you using Jean-Marie?


regards Uwe


So in LyX 1.5.6 is there no way to view and use the Koma-script  
scrlttr2 template directly from within LyX?


Bruce

Dropping a document out of version control

2009-03-23 Thread Shahar Or
Dear list,

How is it possible to stop using version control on a document, please?

Many blessings.
-- 
שחר אור | 050-794 | http://www.shahar-or.co.il
   *** שיעורים פרטיים בלינוקס ותכנה חופשית ***


Selecting empty headings style still creates page numbers

2009-03-23 Thread Shahar Or
Dear list,

Perhaps I'm confused and doing it wrong.

I've selected empty for headings style but still there are page numbers
at the bottom of pages.

Many blessings.
-- 
שחר אור | 050-794 | http://www.shahar-or.co.il
   *** שיעורים פרטיים בלינוקס ותכנה חופשית ***


custom layout -- possible bug?

2009-03-23 Thread Ernesto Posse
Hello. I'm having a bit of an issue with a custom layout I'm trying to
create for ACM SIG proceedings (from the sig-alternate.cls available
at http://www.acm.org/sigs/publications/proceedings-templates)

First I'm creating a layoutfile named sig-alternate.layout and
placed it under my layouts directory. The file contains the following:

--- begin of sig-alternate.layout ---
#% Do not delete the line below; configure depends on this
#  \DeclareLaTeXClass[sig-alternate]{sig-alternate}

Format 11
Input stdclass.inc
--- end of sig-alternate.layout ---

And I create a sample file (test.lyx) with this layout. Then I
generate plain latex from it and this is what I obtain:

--- begin of test.tex ---
%% LyX 1.6.1 created this file.  For more info, see http://www.lyx.org/.
%% Do not edit unless you really know what you are doing.
\documentclass[english]{article}
\usepackage[T1]{fontenc}
\usepackage[latin9]{inputenc}

\usepackage{babel}

\begin{document}

\title{Blah blah}


\author{Me}

\maketitle

Something...
\end{document}
--- end of test.tex ---


The problem is that the first line has the wrong class (article)! Am I
doing something wrong? Why doesn't the generated latex begin with this
\documentclass{sig-alternate}?


Thanks.

PS: I'm running lyx 1.6.1 on ubuntu.

-- 
Ernesto Posse

Applied Formal Methods Group - Software Technology Lab
School of Computing
Queen's University - Kingston, Ontario, Canada


Re: letter?

2009-03-23 Thread Jean-Marie Pacquet

Bruce Pourciau a écrit :


On Mar 22, 2009, at 5:18 PM, Uwe Stöhr wrote:


Jean-Marie Pacquet schrieb:

You should erase the starting comma in the options list of the Latex 
preamble (the one in front of footsepline=true). The new version 
of Koma-script scrlttr2 does not like it.


In LyX 1.6.x the template doesn't have this comma. What LyX version 
are you using Jean-Marie?


regards Uwe


So in LyX 1.5.6 is there no way to view and use the Koma-script 
scrlttr2 template directly from within LyX?


Bruce


Just make sure that the Koma-script options list does not start with a 
comma. Hereunder is a part of the Latex preamble of Koma-script letter2 
template for an old Lyx version (1.4.2):

...
%% THE CLASS OPTIONS
%% Remove preceeding '%' to uncomment an item
\KOMAoptions{%
%,headsepline=true%separate the header with a line on page 1
%,footsepline=true% separate the footer with a line on page 1
%pagenumber=botcenter%   position of the page number (see docu)
%,parskip=false%  Use indent instead of skip (more options cf. docu)
,fromalign=center%alignment of the address
,fromrule=aftername%separate the address with a line?
,fromphone=true% print sender phone number
%,fromfax=true%  print sender fax number
,fromemail=true%   print sender e-mail address
,fromurl=true%   print sender URL
%,fromlogo=true% print a logo (position depends on fromalign)
%,addrfield=false%print an address field?
%,backaddress=false%  print the back address?
%,subject=afteropening,titled% alternative subject layout and position
%,locfield=narrow%  width of the (extra) location field
%,foldmarks=false%  print foldmarks?
%,numericaldate=true%  date layout
%,refline=wide% layout of the refline
}

You will notice that the first uncommented option 
(,fromalign=center%...) starts with a comma). If you try to use this 
template with a more recent Lyx version (in fact a more recent 
Koma-script version) you will get this error:


You have used \KOMAoptions to set `',
but KOMA-Script does not know any option named `'.
See the KOMA-Script manual for more informations about options and
their values.


The solution is to edit the Latex preamble (Menu Document-Parameters 
then Latex preamble) so that the first option reads fromalign=center% 
without the preceding comma.


I understand from Uwe that the new templates for Koma-script letter2 no 
longer have this preceding comma. May be you still have an old template 
with a new version of Koma-script.


Regards
--
jean-marie




Re: Installing biblatex on OSX

2009-03-23 Thread BH
On Mon, Mar 23, 2009 at 11:45 AM, jezZiFeR jezzi...@gmail.com wrote:
 texhash is relevant for updating your TeX installation when you add
 new .sty or .cls packages. It is distinct from reconfiguring LyX,
 which needs to be done when you add new .layout or .module files.

 Thank you for that explanation!


snip

 .layout/.module files are OS agnostic: there is no special Linux or
 Mac or Windows version of them. So you can copy them from one computer
 to another so long as you put them in the right place. But your texmf
 folder is *not* the right place for .layout/.module files;

 YES, that was it! I put it directly into the …/ly 1.6-folder! now it works!

 On my other computer it really have this folder, which I do not have here.
 Is this folder used for LaTex? That´s the way it looks like:

Yes -- everything under texmf is for TeX/BibTeX/LaTeX.

I suspect what confused you was the presence of the lyx folder in
~/Library/texmf/latex. That is not your lyx user's directory (which is
at ~/Library/Application Support/LyX-1.6) but is rather a directory
the LyX Installer creates for its own latex styles and classes. You
should not put anything in that directory, as it may get overwritten
by future versions of the installer.

Bennett


Re: Help for paper about LaTeX/LyX and the meaning of life

2009-03-23 Thread Steve Litt
On Monday 23 March 2009 12:07:00 pm Piero Faustini wrote:
 Hello,
  in a couple of months I have to speak at a conference about computers 
 music/critical editions. I will introduce to a M$Word-enslaved audience the
 great advantages of WYSIWYM (by the way, I'm going to talk about music
 notation software LilyPond too, and the audience will be also
 Finale-enslaved: it's David vs. Goliat).

 I'm a musicologist and I'm not a LaTeX/LyX pro so the thing would
 definitely be something much more like a divulgative/ads/propaganda thing
 than a specialized research. For this reason, in order to give the audience
 some content rather than advertising, I want to cite some statistics,
 relevant opinions, important projects/books/initiatives based on LyX and so
 on, but the LyX site lacks of all of this.

 On the other hand, I would like to introduce the WYSIWYM “philosophy”
 but, as I'm not a semiologist, I don't know where to find some relevant
 thought (or some “effect quotation”!) on the whole
 Content-Form-Structure-and-the-meaning- of-life stuff.

 Any help would be apreciated and - if possible - referenced.

 Thanks, Piero

Hi Piero,

If you haven't already used the word WYSIWYM in your title yet and 
advertised your talk as such, I'd personally use different terminology. I'd 
call it styles based authoring.

LyX is built from the bottom up to make it easy to use (not necessarily create 
or modify, but use) styles. Either character styles or paragraph styles 
(which we LyXers call environments).

Styles-based authoring is a must when writing a long document because it 
promotes consistency. I used the character style myEmph about 30 times last 
night, and every one of them looked identical, both in LyX and in the 
produced PDF. I didn't have to say to myself each time hey, in this book am 
I emphasizing by italicizing, bolding or both? I tell several stories in my 
book, and I like stories to look different from the rest of the text. I don't 
have to, with each new story, ask myself hey, did I italicize stories, 
indent them, shade them, put a box around them, or some combination? No, 
every time I tell a story, I just use my Story environment.

The end result is that my book has a consistency unmatched by people who don't 
use styles-based authoring. Be aware that you can do styles-based authoring 
with MS Word, WordPerfect, of if you're a masochist OpenOffice. But it's 
easier in LyX, and LyX also makes it harder to do one-off formatting of 
characters and paragraphs. We LyXers call such one-off formatting finger 
painting, and the results aren't very good. If one really needs to finger 
paint, that's probably a good indication that what's really needed is pixel 
editor or a vector graphics program.

Some people will tell you LyX is not WYSIWYG. All I know is it's WYSIWYG 
enough that I was able to proofread my book in LyX. A truly non-WYSIWYG would 
require regular recompilation to the finished form to proofread. Wordperfect 
4.x is a non-WYSIWYG example -- an even better one is HTML in a text editor.

Here are some of the reasons I personally use LyX:

* It's rock stable
* It does what you expect it to do
* It turns out VERY good looking text layout
* Its native format is simple to edit with an editor
* Its native format is simple to parse with a program
* Its native format is simple to create with a program
* It supports me with huge community of knowledgeable people
* When I finally write my math book, it will handle equations beautifully

Here are the disadvantages of LyX:

* Creation and modification of custom styles is much harder than MS Word or 
Wordperfect.

If you get ten or twenty more opinions, you'll have a great foundation for 
your presentation.

HTH

SteveT

Steve Litt
Recession Relief Package
http://www.recession-relief.US



Re: Selecting empty headings style still creates page numbers

2009-03-23 Thread M-L
On Mon, 23 Mar 2009 22:03:47 +0200
Shahar Or sha...@shahar-or.co.il wrote:

 Dear list,
 
 Perhaps I'm confused and doing it wrong.
 
 I've selected empty for headings style but still there are page
 numbers at the bottom of pages.
 
 Many blessings.

Try: Document/Settings/Page Layout/Page style = empty

HTH if not maybe a bit more information might make it clearer - eg.
operating system.

That will certainly help.
-- 
Registered Linux User:- 329524
***
The central benefit of Zen, in the context of ordinary ups and downs of
life, is not in preventing the minus and promoting the plus, but in
directing people to the fundamental reality that is not under the sway
of ups and downs. Muso Kokushi
(1275-1351)

***
Debian, just the best way to create magic
___


Re: Smart single quotes in LyX (1.6.1, Mac OS X 10.5.6)

2009-03-23 Thread Grant Jacobs


Thanks for your reply. This doesn't quite address what I was after: 
almost, but not quite.


In case it's of use to others I'm going to explain what I wanted 
again, then how I have a sort-of solution, or more accurately a 
solution that raises another problem in it's place.


I was asking for a means to set up LyX so that whenever I type a 
single quote ('), the single quote would be treated as a smart 
quote, i.e. LyX would interpret it as an open or close quote as 
appropriate.


(I was not trying to set some key sequence to generate a smart single 
quote, I already knew how to do that--that's actually what I wanted 
to *stop* having to do, a key sequence is already set for generating 
a smart single quote by default. I was trying to get a single quote 
to be treated as a smart quote without having to remember to type a 
special key sequence.)


Furthermore, what I really wanted was to still get a straight 
quote, using a key sequence. The overall effect wanted was that of 
having a key sequence that might by default generate a smart quote, 
generate a straight quote and have the quote character default to a 
smart quote. This can be achieved with the double quotes; I was 
trying to achieve the same with a single quote.



If you try set ' to be the mapped key in the fashion BH 
describes, LyX will complain that:


   Shortcut `'' is already bound to:
   self-insert
   You need to remove that binding before creating a new one.

These self-insert bindings seem to be set in a file held within the 
application bundle, so strictly speaking this solution should be 
out of bounds to an end user. Ideally what would nice was a means to 
have ' act as a smart quote through the user interface, and 
hopefully in a way I will not have keep manually updating the binding 
each time I update LyX. This doesn't seem to be possible at present.


My eventual sort-of solution was to edit the latinkeys.bind file (in 
the 'bind' directory of the Resources section of the application), so 
that


   \bind quoteright self-insert

now reads:

   \bind quoteright quote-insert single

This certainly makes all single quotes appear as smart quotes, 
except now I can't find a way to map cmd-' (or whatever) to generate 
a straight quote, as it insists on generating a quote-insert 
single!


While I suspect there will be a solution to this somehow, this does 
suggest to me that the system for applying character mappings has 
some weaknesses.



Grant


On Thu, Mar 19, 2009 at 9:53 PM, Grant Jacobs 
gjac...@bioinfotools.com wrote:


 Is there a means to make smart *single* quotes the default in the same way
 as is done for double quotes?


LyX  Preferences  Editing  Shortcuts

1. Enter quote in the Show key-bindings containing field
2. select quote-insert single
3. click the Modify button
4. click the Clear button on the dialog that pops up
5. type the key you want to bind smart single quotes to
6. click OK
7. click Save

Bennett



--
---
Grant Jacobs Ph.D. BioinfoTools
ph. +64 3 478 0095  (office, after 10am)   PO Box 6129,
or  +64 27 601 5917 (mobile)   Dunedin,
gjac...@bioinfotools.com   NEW ZEALAND.
   Bioinformatics tools: deriving knowledge from biological data  
Bioinformatics tools - software development - consulting - training

 18 years experience in bioinformatics ready to solve your problem
Check out the website for more details: http://www.bioinfotools.com


Re: Help for paper about LaTeX/LyX and the meaning of life

2009-03-23 Thread stefano franchi
2009/3/23 Steve Litt sl...@troubleshooters.com

 On Monday 23 March 2009 12:07:00 pm Piero Faustini wrote:
  Hello,
   in a couple of months I have to speak at a conference about computers 
  music/critical editions. I will introduce to a M$Word-enslaved audience
 the
  great advantages of WYSIWYM (by the way, I'm going to talk about music
  notation software LilyPond too, and the audience will be also
  Finale-enslaved: it's David vs. Goliat).
 
  I'm a musicologist and I'm not a LaTeX/LyX pro so the thing would
  definitely be something much more like a divulgative/ads/propaganda thing
  than a specialized research. For this reason, in order to give the
 audience
  some content rather than advertising, I want to cite some statistics,
  relevant opinions, important projects/books/initiatives based on LyX and
 so
  on, but the LyX site lacks of all of this.
 
  On the other hand, I would like to introduce the WYSIWYM â  philosophyâ
  but, as I'm not a semiologist, I don't know where to find some relevant
  thought (or some â  effect quotationâ  !) on the whole
  Content-Form-Structure-and-the-meaning- of-life stuff.
 
  Any help would be apreciated and - if possible - referenced.
 
  Thanks, Piero

 Hi Piero,

 If you haven't already used the word WYSIWYM in your title yet and
 advertised your talk as such, I'd personally use different terminology. I'd
 call it styles based authoring.

 LyX is built from the bottom up to make it easy to use (not necessarily
 create
 or modify, but use) styles. Either character styles or paragraph styles
 (which we LyXers call environments).

 Styles-based authoring is a must when writing a long document because it
 promotes consistency. I used the character style myEmph about 30 times last
 night, and every one of them looked identical, both in LyX and in the
 produced PDF. I didn't have to say to myself each time hey, in this book
 am
 I emphasizing by italicizing, bolding or both? I tell several stories in my
 book, and I like stories to look different from the rest of the text. I
 don't
 have to, with each new story, ask myself hey, did I italicize stories,
 indent them, shade them, put a box around them, or some combination? No,
 every time I tell a story, I just use my Story environment.

 The end result is that my book has a consistency unmatched by people who
 don't
 use styles-based authoring. Be aware that you can do styles-based authoring
 with MS Word, WordPerfect, of if you're a masochist OpenOffice. But it's
 easier in LyX, and LyX also makes it harder to do one-off formatting of
 characters and paragraphs. We LyXers call such one-off formatting finger
 painting, and the results aren't very good. If one really needs to finger
 paint, that's probably a good indication that what's really needed is pixel
 editor or a vector graphics program.

 Some people will tell you LyX is not WYSIWYG. All I know is it's WYSIWYG
 enough that I was able to proofread my book in LyX. A truly non-WYSIWYG
 would
 require regular recompilation to the finished form to proofread.
 Wordperfect
 4.x is a non-WYSIWYG example -- an even better one is HTML in a text
 editor.

 Here are some of the reasons I personally use LyX:

 * It's rock stable
 * It does what you expect it to do
 * It turns out VERY good looking text layout
 * Its native format is simple to edit with an editor
 * Its native format is simple to parse with a program
 * Its native format is simple to create with a program
 * It supports me with huge community of knowledgeable people
 * When I finally write my math book, it will handle equations beautifully

 Here are the disadvantages of LyX:

 * Creation and modification of custom styles is much harder than MS Word or
 Wordperfect.

 If you get ten or twenty more opinions, you'll have a great foundation for
 your presentation.

 HTH

 SteveT

 Steve Litt
 Recession Relief Package
 http://www.recession-relief.US


Hi Piero,

I'm following Steve's suggestion and posting another opinion.
I have been using LyX exclusively for the last 4 years, and I came to it
from a few years in Framemaker. I used Word before that (since version
1.0...), until it literally ate the first chapter of my dissertation. I have
next to zero coding abilities, and I am an academic in the humanities---a
skill set that may be pretty close to your audience's, I believe.

My reasons for using LyX:

* its file format is text-based and human-readable. That guarantees that my
files will be readable throughout my career---which I hope will last
another  few years. Closed-source formats like MS Word, Framemaker, etc.
expose you to the whims of their manufacturers. That's not so bad in the
private sector, when most of the documents produced are short-lived, but
it's crucial in the academia, when you may want to reuse documents, notes,
etcetera you have written 20 years ago or more. In my case: Adobe
discontinued Framemaker for Linux and for Mac  10.4 (it requires classic).
I now have to keep around (and 

child document not working as advertised?

2009-03-23 Thread Florin Oprina
Hi all.
I use Lyx 1.6.2. I created a child document, selected a default master
document and put all my custom commands in the preamble of the master file.
In the master file I included the child document.

Now, if I compile the child document the settings in the master file seem to
be ignored: the custom commands do not work and bibliographic references are
not resolved.

From the LyX wiki:``Render just a child document and LyX will make sure all
macros are defined correctly, even though their real definition is e.g. in
the master document.''

But I can't get LyX to work as advertised. What am I doing wrong?

Thanks.


«

2009-03-23 Thread John White
I run lyx 1.6.2 on my slackware 12.1 (Vector Linux) system.  I compiled 
it from source.  I use the default CUA bindings which show that pushing 
the quote key (key with " on it" will produce a ".  However, it produces 
a «.  (if this does not come through, its two little <). If I switch to 
aqua bindings, the problem goes away.  However I see no way to edit the 
shortcuts to deal with this.


Any suggestions are appreciated. I have had this problem, if it be such, 
since, I believe, lyx 1.6.0.  I get around it by holding down the SHIFT 
key and then hitting the " key.  However, that is something of a bother.


John




Re: «

2009-03-23 Thread Vincent van Ravesteijn

John White schreef:
I run lyx 1.6.2 on my slackware 12.1 (Vector Linux) system.  I 
compiled it from source.  I use the default CUA bindings which show 
that pushing the quote key (key with " on it" will produce a ".  
However, it produces a «.  (if this does not come through, its two 
little <). If I switch to aqua bindings, the problem goes away.  
However I see no way to edit the shortcuts to deal with this.


Any suggestions are appreciated. I have had this problem, if it be 
such, since, I believe, lyx 1.6.0.  I get around it by holding down 
the SHIFT key and then hitting the " key.  However, that is something 
of a bother.


John



Do you know about the "Document->Settings->Language->Quote Style" option ?

Vincent


Re: «

2009-03-23 Thread John White
No, I did not.  That did the trick. 


Thank you very much!

John

Vincent van Ravesteijn wrote:

John White schreef:
I run lyx 1.6.2 on my slackware 12.1 (Vector Linux) system.  I 
compiled it from source.  I use the default CUA bindings which show 
that pushing the quote key (key with " on it" will produce a ".  
However, it produces a «.  (if this does not come through, its two 
little <). If I switch to aqua bindings, the problem goes away.  
However I see no way to edit the shortcuts to deal with this.


Any suggestions are appreciated. I have had this problem, if it be 
such, since, I believe, lyx 1.6.0.  I get around it by holding down 
the SHIFT key and then hitting the " key.  However, that is something 
of a bother.


John


Do you know about the "Document->Settings->Language->Quote Style" 
option ?


Vincent







Re: letter?

2009-03-23 Thread Jean-Marie Pacquet

Uwe Stöhr a écrit :

Jean-Marie Pacquet schrieb:

You should erase the starting comma in the options list of the Latex 
preamble (the one in front of "footsepline=true"). The new version of 
Koma-script scrlttr2 does not like it.


In LyX 1.6.x the template doesn't have this comma. What LyX version 
are you using Jean-Marie?


regards Uwe
I use various versions (1.4.2 to 1.6.1). The problem of commas in front 
of the options in the Koma-script letter2 template (as it is up to 1.5.4 
I believe) was brought to my attention when someone told me that the 
templates I uploaded to the wiki as french letters examples in July 2008 
were not  working (that was in october 2008).
So when Bruce wrote he had a problem with Koma options in a template, I 
immediatly thought of this. May be it's something different though...


regards
--
jean-marie


Re: letter?

2009-03-23 Thread Helge Hafting

bb wrote:

The Documentation "User Guide" from December 7, 2008 might be outdated?
The Description of writing a letter (p. 50 in the german version of the
document) does not describe the reality. I tried all types of
letter-documents, but none offers the described option of adress-right? 


For instance if I try letter(DIN-Brief, German) I get an environment for
an adressee, but I am unable to write the complete adress with name,
street and town? That is also true for the simple letter environment. 

You can't press enter because that will end the paragraph (and also the 
"address" paragraph type.)


But press ctrl+enter when you need seeveral lines in an address. That 
works. I have written many a letter in LyX. You should really use a 
template, as the letter document class is very fussy about getting most 
of the special paragraph types _and_ in the right order. with a 
template, you just fill out the fields and then it works well.


Helge Hafting


Re: Lilypond integration

2009-03-23 Thread Helge Hafting

Johannes Asal wrote:
I know that you can include Lilypond material by External Material. 
That's ok when you have few notation examples in your text and I 
appreciate that someone took the time to implement it. But I'm trying to 
write a book about jazz harmonics and therefore I need many musical 
examples that have to be changed often in the writing process. The main 
problems I have with the External Material approach are the following:


1. It interrupts the creative process, because you need to prepare the 
lilypond snippets in another editor and import them afterwards


I understand, as I have the same problem with figures. It is possible to 
ease the pain some, though. The external inset has a "edit" button, so 
you can click that in order to edit your music snippet. Newer versions 
of lyx doesn't have the button, instead you right-click the lilypond 
object and select "edit externally" from the menu. So you can "get to 
it" from LyX, but it will obviously still be an external application. 
You may have to define an editor for .ly files in the preferences.


2. You don't have a preview and therefore it is a bit difficult to see 
whether you have written something already or not


Tools->preferences->Graphics, then turn "Instant Preview" ON.
You should now get previews of all external insets, including lilypond 
music.  This is perhaps turned off by default, for performance reasons.


3. If you have to change one of the snippets, you have to find the 
corresponding file


Not really. right-click and select "edit externally" (Or, for older 
versions of lyx: click, then use the edit button in the dialog.)


I don't know if it's possible, but the ideal solution (for me) would be 
some kind of lilypond environment (somewhat like the math environment), 
where you can type in your lilypond code directly. After leaving this 
environment, the graphic is rendered and put into place in the LyX 
document. If you need to change it, you just click on the graphic and 
the code environment pops up again.


This is of course not a simple modification to do, and I guess it 
wouldn't be that fun either, but I'm sure it would be a valueable 
addition to LyX's feature list because as far as I know there is no 
solution today that combines text with music typesetting in a convenient 
and straightforward way. I would even volunteer for implementing parts 
of it if needed.


Support for lilypond-book would be interesting. And sure - if you 
volunteer to do some work then you will get it faster.


This process can be divided into several steps, you won't have to do it 
all at once.


1. Work on the support for lilypond-book.
   This will have LyX create latex output as usual,
   then, add code to run lilypond-book on the produced file
   Finally, let LyX run latex/pdflatex on the file produced, as usual.
   So this step is mostly about inserting an invocation
   of "lilypond-book" into the existing workflow.

   At this point, use a new document settings to decide whether
   lyx should do this lilypond-book step or not. Many lyx documents
   have no lilypond content, and many lyx users don't have lilypond.

   Use the latex inset for lilypond code in the beginning.
   To enter music, type stuff like
\begin{lilypond} { c d e } \end{lilypond}
   There will be no preview so far.

2. With lilypond-book working, start on the lilypond environment.
   first figure out if it ought to be a inset, a layout module, or
   whatever. I guess an inset will be the way.

   Then add the rule about how LyX produces latex code when processing
   a lilypond inset. This is easy - just output \begin{lilypond}
   followed by the inset contents, followed by \end{lilypond}

   At this point, you have working lilypond-book!

3. Some fine-tuning. Remove the document setting that specifies if this
   is a lilypond document. Instead, add logic so that LyX will run
   lilypond-book whenever the document contains a lilypond inset.
   You can perhaps make use of similiar logic that add latex packages
   on an as-needed basis.

4. The preview. Arrange so Lyx runs the inset content through
   lilypond, and display the resulting .png if this succeeds.
   I am not sure how to do this part, but look at how it is done
   for math and graphichs insets.


Helge Hafting


Re: Installing biblatex on OSX

2009-03-23 Thread jezZiFeR
Hello,

 I have a problem with installing bibtext on a second OSX-Computer. I put
> the
> > biblatex.module-file directly into the …/library/Lyx-1.6-Folder. Maybe
> this
> > is the problem, but I think this is what the wiki prompts me to do. I
> > reconfigured Lyx various times without succes – it does not find the
> module.
>
> Did you put it in ~/Library/Application Support/LyX-1.6/layout folder?
> (That's where it belongs.)


Yes, that´s where I have put it…


> > Running texhas I get error-messages for every file in the texmf-folder,
> for
> > example:
> >>
> >> texhash: /usr/local/texlive/2008/texmf-var: directory not writable.
> >> Skipping...
>
> This indicates you need to run texhash as root: sudo texhash.


I did that now, but Lyx still does not find the biblatex-module.


> On my other computer I have a texmf-folder
> > directly in the library, where the biblatex.module is saved, and
> everything
> > works fine. But on the other computer this folder does not exist…
>
> It won't exist unless you create it.


Do I need that folder or is it okay, that biblatex-module is in
…/library/applicationsupport/lyx 1.6 ? Where do all the other files lie on
my computer, on which I don´t have that texmf-folfer directly in the
library?

Could I just copy that folder from one computer to the other?

Thank you, best
Jess


Re: Installing biblatex on OSX

2009-03-23 Thread BH
On Mon, Mar 23, 2009 at 10:40 AM, jezZiFeR  wrote:
> Hello,
>
>  I have a problem with installing bibtext on a second OSX-Computer. I put
>> the
>> > biblatex.module-file directly into the …/library/Lyx-1.6-Folder. Maybe
>> this
>> > is the problem, but I think this is what the wiki prompts me to do. I
>> > reconfigured Lyx various times without succes – it does not find the
>> module.
>>
>> Did you put it in ~/Library/Application Support/LyX-1.6/layout folder?
>> (That's where it belongs.)
>
>
> Yes, that´s where I have put it…

Then it should be recognized -- but only after you reconfigure LyX
(LyX > Reconfigure).

>> > Running texhas I get error-messages for every file in the texmf-folder,
>> for
>> > example:
>> >>
>> >> texhash: /usr/local/texlive/2008/texmf-var: directory not writable.
>> >> Skipping...
>>
>> This indicates you need to run texhash as root: sudo texhash.
>
>
> I did that now, but Lyx still does not find the biblatex-module.

texhash is relevant for updating your TeX installation when you add
new .sty or .cls packages. It is distinct from reconfiguring LyX,
which needs to be done when you add new .layout or .module files.

>> On my other computer I have a texmf-folder
>> > directly in the library, where the biblatex.module is saved, and
>> everything
>> > works fine. But on the other computer this folder does not exist…
>>
>> It won't exist unless you create it.
>
>
> Do I need that folder or is it okay, that biblatex-module is in
> …/library/applicationsupport/lyx 1.6 ? Where do all the other files lie on
> my computer, on which I don´t have that texmf-folfer directly in the
> library?
>
> Could I just copy that folder from one computer to the other?

.layout/.module files are OS agnostic: there is no special Linux or
Mac or Windows version of them. So you can copy them from one computer
to another so long as you put them in the right place. But your texmf
folder is *not* the right place for .layout/.module files; those
belong in your LyX User's directory (which on Mac is
~/Library/Application Support/LyX-1.6) in the layout subdir.

Please make sure when you're reporting your problems that you are
accurately describing what folders you are putting files in. There is
no .../library/applicationsupport/lyx 1.6 folder; it's
.../Library/Application Support/LyX-1.6 (note capitalization, spacing,
and hypthen). If you don't accurately describe it here, we're unsure
whether you're actually putting in in the right place or not, and so
unsure whether the problem is user error or a bug.

Bennett


Re: Lilypond integration

2009-03-23 Thread Piero Faustini

> Support for lilypond-book would be interesting. And sure - if you 
> volunteer to do some work then you will get it faster.

I would happily contribute, but I'm not a programmer. May I be of some help 
just giving some suggestion or testing?


> This process can be divided into several steps, you won't have to do it 
> all at once.

I think your 4-steps are a perfect roadmap.

I have a doubt, although it's regarding more the LaTeX-LilyPond integration: 
lily-book knows how to break music lines (or pages) in a given latex 
environment, but it can't guess if this environment will change after LaTeX 
compile. I mean: I'm writing a komascript book using some packages which can 
alter heavily the shape of the text, such as BibLaTeX, by adding more text. 
This is the reason why my LyX's LaTeX run up to 4 times when compiling, and 
everything works ok. Can lilypond-book give instructions at the beginning of 
this process? Can it be run more than once between Latex runs just to refine 
the process? If yes, what about LyX and all this mess?

Anyway, to all: keep me updated about any Lilypond-book integration attempt.
Piero



Re: Installing biblatex on OSX

2009-03-23 Thread jezZiFeR

Then it should be recognized -- but only after you reconfigure LyX

(LyX > Reconfigure).


I did that various times, but it still does not work.


Running texhas I get error-messages for every file in the texmf- 
folder,

for

example:


texhash: /usr/local/texlive/2008/texmf-var: directory not  
writable.

Skipping...


This indicates you need to run texhash as root: sudo texhash.



I did that now, but Lyx still does not find the biblatex-module.


texhash is relevant for updating your TeX installation when you add
new .sty or .cls packages. It is distinct from reconfiguring LyX,
which needs to be done when you add new .layout or .module files.


Thank you for that explanation!


On my other computer I have a texmf-folder

directly in the library, where the biblatex.module is saved, and

everything

works fine. But on the other computer this folder does not exist…


It won't exist unless you create it.



Do I need that folder or is it okay, that biblatex-module is in
…/library/applicationsupport/lyx 1.6 ? Where do all the other files  
lie on

my computer, on which I don´t have that texmf-folfer directly in the
library?

Could I just copy that folder from one computer to the other?


.layout/.module files are OS agnostic: there is no special Linux or
Mac or Windows version of them. So you can copy them from one computer
to another so long as you put them in the right place. But your texmf
folder is *not* the right place for .layout/.module files;


YES, that was it! I put it directly into the …/ly 1.6-folder! now it  
works!


On my other computer it really have this folder, which I do not have  
here. Is this folder used for LaTex? That´s the way it looks like:
<>




those
belong in your LyX User's directory (which on Mac is
~/Library/Application Support/LyX-1.6) in the layout subdir.

Please make sure when you're reporting your problems that you are
accurately describing what folders you are putting files in. There is
no .../library/applicationsupport/lyx 1.6 folder; it's
.../Library/Application Support/LyX-1.6 (note capitalization, spacing,
and hypthen). If you don't accurately describe it here, we're unsure
whether you're actually putting in in the right place or not, and so
unsure whether the problem is user error or a bug.


I see, thank you. Now I have put it in .../Library/Application Support/ 
LyX-1.6/layouts


Thank you a lot!
Jess*

Help for paper about LaTeX/LyX and the meaning of life

2009-03-23 Thread Piero Faustini
Hello,
 in a couple of months I have to speak at a conference about computers & 
music/critical editions. I will introduce to a M$Word-enslaved audience the 
great advantages of WYSIWYM (by the way, I'm going to talk about music notation 
software LilyPond too, and the audience will be also Finale-enslaved: it's 
David vs. Goliat).

I'm a musicologist and I'm not a LaTeX/LyX pro so the thing would definitely be 
something much more like a divulgative/ads/propaganda thing than a specialized 
research. For this reason, in order to give the audience some "content" rather 
than advertising, I want to cite some statistics, relevant opinions, important 
projects/books/initiatives based on LyX and so on, but the LyX site lacks of 
all of this.

On the other hand, I would like to introduce the WYSIWYM “philosophy” but, as 
I'm not a semiologist, I don't know where to find some relevant thought (or 
some “effect quotation”!) on the whole Content-Form-Structure-and-the-meaning-
of-life stuff.

Any help would be apreciated and - if possible - referenced.

Thanks, Piero



Re: Help for paper about LaTeX/LyX and the meaning of life

2009-03-23 Thread Les Denham
On Monday 23 March 2009, Piero Faustini wrote:
> Any help would be apreciated and - if possible - referenced.

Piero,

Here are some on-line resources:
http://www.lyx.org/PressAboutLyX
http://www.linux.com/feature/56471
http://www.linuxjournal.com/article/9085


-- 
Les

~~
Please avoid sending me Word or PowerPoint attachments.
See http://www.gnu.org/philosophy/no-word-attachments.html


Re: letter?

2009-03-23 Thread Bruce Pourciau


On Mar 22, 2009, at 5:18 PM, Uwe Stöhr wrote:


Jean-Marie Pacquet schrieb:

You should erase the starting comma in the options list of the  
Latex preamble (the one in front of "footsepline=true"). The new  
version of Koma-script scrlttr2 does not like it.


In LyX 1.6.x the template doesn't have this comma. What LyX version  
are you using Jean-Marie?


regards Uwe


So in LyX 1.5.6 is there no way to view and use the Koma-script  
scrlttr2 template directly from within LyX?


Bruce

Dropping a document out of version control

2009-03-23 Thread Shahar Or
Dear list,

How is it possible to stop using version control on a document, please?

Many blessings.
-- 
שחר אור | 050-794 | http://www.shahar-or.co.il
   *** שיעורים פרטיים בלינוקס ותכנה חופשית ***


Selecting empty headings style still creates page numbers

2009-03-23 Thread Shahar Or
Dear list,

Perhaps I'm confused and doing it wrong.

I've selected empty for headings style but still there are page numbers
at the bottom of pages.

Many blessings.
-- 
שחר אור | 050-794 | http://www.shahar-or.co.il
   *** שיעורים פרטיים בלינוקס ותכנה חופשית ***


custom layout -- possible bug?

2009-03-23 Thread Ernesto Posse
Hello. I'm having a bit of an issue with a custom layout I'm trying to
create for ACM SIG proceedings (from the sig-alternate.cls available
at http://www.acm.org/sigs/publications/proceedings-templates)

First I'm creating a layoutfile named "sig-alternate.layout" and
placed it under my layouts directory. The file contains the following:

--- begin of sig-alternate.layout ---
#% Do not delete the line below; configure depends on this
#  \DeclareLaTeXClass[sig-alternate]{sig-alternate}

Format 11
Input stdclass.inc
--- end of sig-alternate.layout ---

And I create a sample file (test.lyx) with this layout. Then I
generate plain latex from it and this is what I obtain:

--- begin of test.tex ---
%% LyX 1.6.1 created this file.  For more info, see http://www.lyx.org/.
%% Do not edit unless you really know what you are doing.
\documentclass[english]{article}
\usepackage[T1]{fontenc}
\usepackage[latin9]{inputenc}

\usepackage{babel}

\begin{document}

\title{Blah blah}


\author{Me}

\maketitle

Something...
\end{document}
--- end of test.tex ---


The problem is that the first line has the wrong class (article)! Am I
doing something wrong? Why doesn't the generated latex begin with this
\documentclass{sig-alternate}?


Thanks.

PS: I'm running lyx 1.6.1 on ubuntu.

-- 
Ernesto Posse

Applied Formal Methods Group - Software Technology Lab
School of Computing
Queen's University - Kingston, Ontario, Canada


Re: letter?

2009-03-23 Thread Jean-Marie Pacquet

Bruce Pourciau a écrit :


On Mar 22, 2009, at 5:18 PM, Uwe Stöhr wrote:


Jean-Marie Pacquet schrieb:

You should erase the starting comma in the options list of the Latex 
preamble (the one in front of "footsepline=true"). The new version 
of Koma-script scrlttr2 does not like it.


In LyX 1.6.x the template doesn't have this comma. What LyX version 
are you using Jean-Marie?


regards Uwe


So in LyX 1.5.6 is there no way to view and use the Koma-script 
scrlttr2 template directly from within LyX?


Bruce


Just make sure that the Koma-script options list does not start with a 
comma. Hereunder is a part of the Latex preamble of Koma-script letter2 
template for an old Lyx version (1.4.2):

...
%% THE CLASS OPTIONS
%% Remove preceeding '%' to uncomment an item
\KOMAoptions{%
%,headsepline=true%separate the header with a line on page >1
%,footsepline=true% separate the footer with a line on page >1
%pagenumber=botcenter%   position of the page number (see docu)
%,parskip=false%  Use indent instead of skip (more options cf. docu)
,fromalign=center%alignment of the address
,fromrule=aftername%separate the address with a line?
,fromphone=true% print sender phone number
%,fromfax=true%  print sender fax number
,fromemail=true%   print sender e-mail address
,fromurl=true%   print sender URL
%,fromlogo=true% print a logo (position depends on fromalign)
%,addrfield=false%print an address field?
%,backaddress=false%  print the back address?
%,subject=afteropening,titled% alternative subject layout and position
%,locfield=narrow%  width of the (extra) location field
%,foldmarks=false%  print foldmarks?
%,numericaldate=true%  date layout
%,refline=wide% layout of the refline
}

You will notice that the first uncommented option 
(",fromalign=center%...") starts with a comma). If you try to use this 
template with a more recent Lyx version (in fact a more recent 
Koma-script version) you will get this error:


You have used \KOMAoptions to set `',
but KOMA-Script does not know any option named `'.
See the KOMA-Script manual for more informations about options and
their values.


The solution is to edit the Latex preamble (Menu Document->Parameters 
then Latex preamble) so that the first option reads "fromalign=center%" 
without the preceding comma.


I understand from Uwe that the new templates for Koma-script letter2 no 
longer have this preceding comma. May be you still have an old template 
with a new version of Koma-script.


Regards
--
jean-marie




Re: Installing biblatex on OSX

2009-03-23 Thread BH
On Mon, Mar 23, 2009 at 11:45 AM, jezZiFeR  wrote:
>> texhash is relevant for updating your TeX installation when you add
>> new .sty or .cls packages. It is distinct from reconfiguring LyX,
>> which needs to be done when you add new .layout or .module files.
>
> Thank you for that explanation!
>



>> .layout/.module files are OS agnostic: there is no special Linux or
>> Mac or Windows version of them. So you can copy them from one computer
>> to another so long as you put them in the right place. But your texmf
>> folder is *not* the right place for .layout/.module files;
>
> YES, that was it! I put it directly into the …/ly 1.6-folder! now it works!
>
> On my other computer it really have this folder, which I do not have here.
> Is this folder used for LaTex? That´s the way it looks like:

Yes -- everything under texmf is for TeX/BibTeX/LaTeX.

I suspect what confused you was the presence of the "lyx" folder in
~/Library/texmf/latex. That is not your lyx user's directory (which is
at ~/Library/Application Support/LyX-1.6) but is rather a directory
the LyX Installer creates for its own latex styles and classes. You
should not put anything in that directory, as it may get overwritten
by future versions of the installer.

Bennett


Re: Help for paper about LaTeX/LyX and the meaning of life

2009-03-23 Thread Steve Litt
On Monday 23 March 2009 12:07:00 pm Piero Faustini wrote:
> Hello,
>  in a couple of months I have to speak at a conference about computers &
> music/critical editions. I will introduce to a M$Word-enslaved audience the
> great advantages of WYSIWYM (by the way, I'm going to talk about music
> notation software LilyPond too, and the audience will be also
> Finale-enslaved: it's David vs. Goliat).
>
> I'm a musicologist and I'm not a LaTeX/LyX pro so the thing would
> definitely be something much more like a divulgative/ads/propaganda thing
> than a specialized research. For this reason, in order to give the audience
> some "content" rather than advertising, I want to cite some statistics,
> relevant opinions, important projects/books/initiatives based on LyX and so
> on, but the LyX site lacks of all of this.
>
> On the other hand, I would like to introduce the WYSIWYM “philosophy”
> but, as I'm not a semiologist, I don't know where to find some relevant
> thought (or some “effect quotation”!) on the whole
> Content-Form-Structure-and-the-meaning- of-life stuff.
>
> Any help would be apreciated and - if possible - referenced.
>
> Thanks, Piero

Hi Piero,

If you haven't already used the word "WYSIWYM" in your title yet and 
advertised your talk as such, I'd personally use different terminology. I'd 
call it "styles based authoring."

LyX is built from the bottom up to make it easy to use (not necessarily create 
or modify, but use) styles. Either character styles or paragraph styles 
(which we LyXers call "environments").

Styles-based authoring is a must when writing a long document because it 
promotes consistency. I used the character style myEmph about 30 times last 
night, and every one of them looked identical, both in LyX and in the 
produced PDF. I didn't have to say to myself each time "hey, in this book am 
I emphasizing by italicizing, bolding or both? I tell several stories in my 
book, and I like stories to look different from the rest of the text. I don't 
have to, with each new story, ask myself "hey, did I italicize stories, 
indent them, shade them, put a box around them, or some combination?" No, 
every time I tell a story, I just use my Story environment.

The end result is that my book has a consistency unmatched by people who don't 
use styles-based authoring. Be aware that you can do styles-based authoring 
with MS Word, WordPerfect, of if you're a masochist OpenOffice. But it's 
easier in LyX, and LyX also makes it harder to do one-off formatting of 
characters and paragraphs. We LyXers call such one-off formatting "finger 
painting", and the results aren't very good. If one really needs to finger 
paint, that's probably a good indication that what's really needed is pixel 
editor or a vector graphics program.

Some people will tell you LyX is not WYSIWYG. All I know is it's WYSIWYG 
enough that I was able to proofread my book in LyX. A truly non-WYSIWYG would 
require regular recompilation to the finished form to proofread. Wordperfect 
4.x is a non-WYSIWYG example -- an even better one is HTML in a text editor.

Here are some of the reasons I personally use LyX:

* It's rock stable
* It does what you expect it to do
* It turns out VERY good looking text layout
* Its native format is simple to edit with an editor
* Its native format is simple to parse with a program
* Its native format is simple to create with a program
* It supports me with huge community of knowledgeable people
* When I finally write my math book, it will handle equations beautifully

Here are the disadvantages of LyX:

* Creation and modification of custom styles is much harder than MS Word or 
Wordperfect.

If you get ten or twenty more opinions, you'll have a great foundation for 
your presentation.

HTH

SteveT

Steve Litt
Recession Relief Package
http://www.recession-relief.US



Re: Selecting empty headings style still creates page numbers

2009-03-23 Thread M-L
On Mon, 23 Mar 2009 22:03:47 +0200
Shahar Or  wrote:

> Dear list,
> 
> Perhaps I'm confused and doing it wrong.
> 
> I've selected empty for headings style but still there are page
> numbers at the bottom of pages.
> 
> Many blessings.

Try: Document/Settings/Page Layout/Page style = empty

HTH if not maybe a bit more information might make it clearer - eg.
operating system.

That will certainly help.
-- 
Registered Linux User:- 329524
***
The central benefit of Zen, in the context of ordinary ups and downs of
life, is not in preventing the minus and promoting the plus, but in
directing people to the fundamental reality that is not under the sway
of ups and downs. Muso Kokushi
(1275-1351)

***
Debian, just the best way to create magic
___


Re: Smart single quotes in LyX (1.6.1, Mac OS X 10.5.6)

2009-03-23 Thread Grant Jacobs


Thanks for your reply. This doesn't quite address what I was after: 
almost, but not quite.


In case it's of use to others I'm going to explain what I wanted 
again, then how I have a sort-of solution, or more accurately a 
solution that raises another problem in it's place.


I was asking for a means to set up LyX so that whenever I type a 
single quote ('), the single quote would be treated as a "smart" 
quote, i.e. LyX would interpret it as an open or close quote as 
appropriate.


(I was not trying to set some key sequence to generate a smart single 
quote, I already knew how to do that--that's actually what I wanted 
to *stop* having to do, a key sequence is already set for generating 
a "smart" single quote by default. I was trying to get a single quote 
to be treated as a smart quote without having to remember to type a 
"special" key sequence.)


Furthermore, what I really wanted was to still get a "straight" 
quote, using a key sequence. The overall effect wanted was that of 
having a key sequence that might by default generate a "smart quote", 
generate a straight quote and have the quote character default to a 
smart quote. This can be achieved with the double quotes; I was 
trying to achieve the same with a single quote.



If you try set "'" to be the "mapped" key in the fashion BH 
describes, LyX will complain that:


   Shortcut `'' is already bound to:
   self-insert
   You need to remove that binding before creating a new one.

These "self-insert" bindings seem to be set in a file held within the 
application bundle, so "strictly speaking" this solution should be 
out of bounds to an end user. Ideally what would nice was a means to 
have "'" act as a "smart quote" through the user interface, and 
hopefully in a way I will not have keep manually updating the binding 
each time I update LyX. This doesn't seem to be possible at present.


My eventual sort-of solution was to edit the latinkeys.bind file (in 
the 'bind' directory of the Resources section of the application), so 
that


   \bind "quoteright" "self-insert"

now reads:

   \bind "quoteright" "quote-insert single"

This certainly makes all single quotes appear as "smart quotes", 
except now I can't find a way to map cmd-' (or whatever) to generate 
a "straight" quote, as it insists on generating a "quote-insert 
single"!


While I suspect there will be a solution to this somehow, this does 
suggest to me that the system for applying character mappings has 
some weaknesses.



Grant


On Thu, Mar 19, 2009 at 9:53 PM, Grant Jacobs 
 wrote:


 Is there a means to make "smart" *single* quotes the default in the same way
 as is done for double quotes?


LyX > Preferences > Editing > Shortcuts

1. Enter "quote" in the "Show key-bindings containing" field
2. select "quote-insert single"
3. click the "Modify button"
4. click the "Clear" button on the dialog that pops up
5. type the key you want to bind smart single quotes to
6. click "OK"
7. click "Save"

Bennett



--
---
Grant Jacobs Ph.D. BioinfoTools
ph. +64 3 478 0095  (office, after 10am)   PO Box 6129,
or  +64 27 601 5917 (mobile)   Dunedin,
gjac...@bioinfotools.com   NEW ZEALAND.
   Bioinformatics tools: deriving knowledge from biological data  
Bioinformatics tools - software development - consulting - training

 18 years experience in bioinformatics ready to solve your problem
Check out the website for more details: http://www.bioinfotools.com


Re: Help for paper about LaTeX/LyX and the meaning of life

2009-03-23 Thread stefano franchi
2009/3/23 Steve Litt 

> On Monday 23 March 2009 12:07:00 pm Piero Faustini wrote:
> > Hello,
> >  in a couple of months I have to speak at a conference about computers &
> > music/critical editions. I will introduce to a M$Word-enslaved audience
> the
> > great advantages of WYSIWYM (by the way, I'm going to talk about music
> > notation software LilyPond too, and the audience will be also
> > Finale-enslaved: it's David vs. Goliat).
> >
> > I'm a musicologist and I'm not a LaTeX/LyX pro so the thing would
> > definitely be something much more like a divulgative/ads/propaganda thing
> > than a specialized research. For this reason, in order to give the
> audience
> > some "content" rather than advertising, I want to cite some statistics,
> > relevant opinions, important projects/books/initiatives based on LyX and
> so
> > on, but the LyX site lacks of all of this.
> >
> > On the other hand, I would like to introduce the WYSIWYM â  philosophyâ
> > but, as I'm not a semiologist, I don't know where to find some relevant
> > thought (or some â  effect quotationâ  !) on the whole
> > Content-Form-Structure-and-the-meaning- of-life stuff.
> >
> > Any help would be apreciated and - if possible - referenced.
> >
> > Thanks, Piero
>
> Hi Piero,
>
> If you haven't already used the word "WYSIWYM" in your title yet and
> advertised your talk as such, I'd personally use different terminology. I'd
> call it "styles based authoring."
>
> LyX is built from the bottom up to make it easy to use (not necessarily
> create
> or modify, but use) styles. Either character styles or paragraph styles
> (which we LyXers call "environments").
>
> Styles-based authoring is a must when writing a long document because it
> promotes consistency. I used the character style myEmph about 30 times last
> night, and every one of them looked identical, both in LyX and in the
> produced PDF. I didn't have to say to myself each time "hey, in this book
> am
> I emphasizing by italicizing, bolding or both? I tell several stories in my
> book, and I like stories to look different from the rest of the text. I
> don't
> have to, with each new story, ask myself "hey, did I italicize stories,
> indent them, shade them, put a box around them, or some combination?" No,
> every time I tell a story, I just use my Story environment.
>
> The end result is that my book has a consistency unmatched by people who
> don't
> use styles-based authoring. Be aware that you can do styles-based authoring
> with MS Word, WordPerfect, of if you're a masochist OpenOffice. But it's
> easier in LyX, and LyX also makes it harder to do one-off formatting of
> characters and paragraphs. We LyXers call such one-off formatting "finger
> painting", and the results aren't very good. If one really needs to finger
> paint, that's probably a good indication that what's really needed is pixel
> editor or a vector graphics program.
>
> Some people will tell you LyX is not WYSIWYG. All I know is it's WYSIWYG
> enough that I was able to proofread my book in LyX. A truly non-WYSIWYG
> would
> require regular recompilation to the finished form to proofread.
> Wordperfect
> 4.x is a non-WYSIWYG example -- an even better one is HTML in a text
> editor.
>
> Here are some of the reasons I personally use LyX:
>
> * It's rock stable
> * It does what you expect it to do
> * It turns out VERY good looking text layout
> * Its native format is simple to edit with an editor
> * Its native format is simple to parse with a program
> * Its native format is simple to create with a program
> * It supports me with huge community of knowledgeable people
> * When I finally write my math book, it will handle equations beautifully
>
> Here are the disadvantages of LyX:
>
> * Creation and modification of custom styles is much harder than MS Word or
> Wordperfect.
>
> If you get ten or twenty more opinions, you'll have a great foundation for
> your presentation.
>
> HTH
>
> SteveT
>
> Steve Litt
> Recession Relief Package
> http://www.recession-relief.US
>
>
Hi Piero,

I'm following Steve's suggestion and posting another opinion.
I have been using LyX exclusively for the last 4 years, and I came to it
from a few years in Framemaker. I used Word before that (since version
1.0...), until it literally ate the first chapter of my dissertation. I have
next to zero coding abilities, and I am an academic in the humanities---a
skill set that may be pretty close to your audience's, I believe.

My reasons for using LyX:

* its file format is text-based and human-readable. That guarantees that my
files will be readable throughout my career---which I hope will last
another  few years. Closed-source formats like MS Word, Framemaker, etc.
expose you to the whims of their manufacturers. That's not so bad in the
private sector, when most of the documents produced are short-lived, but
it's crucial in the academia, when you may want to reuse documents, notes,
etcetera you have written 20 years ago or 

child document not working as advertised?

2009-03-23 Thread Florin Oprina
Hi all.
I use Lyx 1.6.2. I created a child document, selected a default master
document and put all my custom commands in the preamble of the master file.
In the master file I included the child document.

Now, if I compile the child document the settings in the master file seem to
be ignored: the custom commands do not work and bibliographic references are
not resolved.

>From the LyX wiki:``Render just a child document and LyX will make sure all
macros are defined correctly, even though their real definition is e.g. in
the master document.''

But I can't get LyX to work as advertised. What am I doing wrong?

Thanks.